K
Khách

Hãy nhập câu hỏi của bạn vào đây, nếu là tài khoản VIP, bạn sẽ được ưu tiên trả lời.

5 tháng 7 2017

Ta có:\(A=\left(1-\frac{1}{2}\right)\times\left(1-\frac{1}{3}\right)\times.......\times\left(1-\frac{1}{2000}\right)\)

\(=\frac{1}{2}\times\frac{2}{3}\times\frac{3}{4}\times..........\times\frac{1999}{2000}\)

\(=\frac{1}{2000}\)

Vậy giá trị của A là:\(\frac{1}{2000}\)

22 tháng 7 2017

@@ gửi ít thôi bạn

23 tháng 7 2017

bạn lm từng bài cg đc mà

a: \(=\dfrac{3}{2}\cdot\dfrac{4}{3}\cdot...\cdot\dfrac{2001}{2000}=\dfrac{2001}{2}\)

b: \(=10.1\left(340+120-270\right)=10.1\cdot190=101\cdot19=1919\)

c: \(=24\%+8\%+59\%+9\%=1\)

13 tháng 12 2022

Bài 2:

\(A=\dfrac{x\left(x^3+1\right)}{x^2-x+1}-\dfrac{x\left(x^3-1\right)}{x^2+x+1}\)

\(=x\left(x+1\right)-x\left(x-1\right)\)

=x^2+x-x^2+x

=2x

9 tháng 7 2019

(Không biết là dấu // của bạn là gì có phải | giá trị tuyệt đối?)

1, Không có giá trị lớn nhấn vì số mũ dương. Giá trị nhỏ nhất là 2019. x=1; y=2

2, Không có giá trị lớn nhất), Giá trị nhỏ nhất tại: (vì giá trị tuyệt đối luôn dương)

https://hotavn.ga/horobot/horobotmath.php?s=Tra+t%C6%B0%CC%80&val=min(%7Cx%2B3%7C%2B%7Cx-y%2B4%7C-10)

3, C <= 2000 vì (giá trị tuyệt đối luôn dương mà đằng trước dấu giá trị tuyệt đối là - nên luôn âm)
=> 

4, vì số mũ dương mà ta lại có 2 ẩn trong đó một ẩn luôn dương và một ẩn luôn âm nên không có giá trị lớn nhất và nhỏ nhất
 

9 tháng 7 2019

1, Ta có: (x - 1)2000 \(\ge\)\(\forall\)x

|y - 2|2000 \(\ge\)\(\forall\)y

=> (x - 1)2000 + |y - 2|2000 + 2019 \(\ge\)2019 \(\forall\)x, y

hay A \(\ge\)2019 \(\forall\)x,y

Dấu "=" xảy ra khi: \(\hept{\begin{cases}x-1=0\\y-2=0\end{cases}}\) <=> \(\hept{\begin{cases}x=1\\y=2\end{cases}}\)

Vậy Amin = 2019 tại  x = 1 và y = 2

2) Ta có: |x + 3| \(\ge\)\(\forall\)x

|x - y + 4| \(\ge\) 0 \(\forall\)x, y

=> |x + 3| + |x - y + 4| - 10 \(\ge\)-10  \(\forall\)x,y

hay B \(\ge\)-10 \(\forall\)x,y

Dấu "=" xảy ra khi: \(\hept{\begin{cases}x+3=0\\x-y+4=0\end{cases}}\) <=> \(\hept{\begin{cases}x=-3\\x-y=-4\end{cases}}\) <=> \(\hept{\begin{cases}x=-3\\y=1\end{cases}}\)

vậy Bmin = -10 tại x = -3  và y = 1

a: \(=\dfrac{3}{2}\cdot\dfrac{4}{3}\cdot...\cdot\dfrac{2001}{2000}=\dfrac{2001}{2}\)

b: \(=101\left(34+13-27\right)=101\cdot20=2020\)

c: \(=24\%+8\%+59\%+9\%=1\)

19 tháng 2 2022

1000,5

19 tháng 2 2022

\(\left(\dfrac{2+1}{2}\right)\left(\dfrac{3+1}{3}\right)\left(\dfrac{4+1}{4}\right)...\left(\dfrac{2000+1}{2000}\right)\)

\(=\dfrac{3.4.5...2001}{2.3.4...2000}=\dfrac{2001}{2}\)

19 tháng 2 2022

Ta có:A=(1−12)×(1−13)×.......×(1−12000)A=(1−12)×(1−13)×.......×(1−12000)

=12×23×34×..........×19992000=12×23×34×..........×19992000

=12000=12000

Vậy giá trị của A là:12000

\(=\dfrac{3}{2}\cdot\dfrac{4}{3}\cdot...\cdot\dfrac{2001}{2000}=\dfrac{2001}{2}\)

23 tháng 7 2017

\(\dfrac{1}{\left(x+1\right)\left(x+2\right)}+\dfrac{1}{\left(x+2\right)\left(x+3\right)}+\dfrac{1}{\left(x+3\right)\left(x+4\right)}+...+\dfrac{1}{\left(x+1999\right)\left(x+2000\right)}=\dfrac{1}{x+200}=\dfrac{1}{5}\)\(\Rightarrow\dfrac{1}{x+1}-\dfrac{1}{x+2}+\dfrac{1}{x+2}-\dfrac{1}{x+3}+\dfrac{1}{x+3}-\dfrac{1}{x+4}+...+\dfrac{1}{x+1999}-\dfrac{1}{x-2000}=\dfrac{1}{x+200}=\dfrac{1}{5}\)

\(\Rightarrow\dfrac{1}{x+1}-\dfrac{1}{x+2000}=\dfrac{1}{x+200}=\dfrac{1}{5}\)

Đề này sai nhé,hình như thừa dữ kiện đề r